Difference between revisions of "2016 AMC 12A Problems/Problem 3"

(Solution)
(Redirect to matching 10A problem)
 
(3 intermediate revisions by one other user not shown)
Line 1: Line 1:
 
+
#REDIRECT [[2016 AMC 10A Problems/Problem 4]]
 
 
==Solution==
 
<cmath>\text{rem}\left(\frac{3}{8},-\frac{2}{5}\right)</cmath>
 
<cmath>=\frac{3}{8}-\left(-\frac{2}{5}\right)\Big\lfloor\frac{\frac{3}{8}}{-\frac{2}{5}}\Big\rfloor</cmath>
 
<cmath>=\frac{3}{8}+\left(\frac{2}{5}\right)\Big\lfloor -\frac{15}{16}\Big\rfloor</cmath>
 
<cmath>=\frac{3}{8}+\left(\frac{2}{5}\right)\left(-1\right)</cmath>
 
<cmath>=\frac{3}{8}-\frac{2}{5}</cmath>
 
<cmath>=\boxed{\textbf{(B)}-\frac{1}{40}}</cmath>
 

Latest revision as of 12:51, 4 February 2016